×

UPSC Courses

UPSC Previous Year Solved Papers

INDIAN POLITY & GOVERNANCE

1. The Dinesh Goswami Committee recommended

Explanation: A. In 1990, Dinesh Goswami Committee recommended the State funding of Parliamentary Elections.

2. Which of the following is NOT a Principle of Panch-sheel?

Explanation: A. The principles of Panchsheel are: 1. Mutual respect for each Others territorial integrity and sovereignty, 2. Non-Aggression, 3. Non-Interference in each other's internal affairs, 4.Peaceful Coexistence, 5. Equality and mutual Benefit.

3. In which of the following countries will the no-confidence motion to bring down the government passed by the legislature be valid only when the legislature is able to find simultaneously a majority to elect a successor government ?

Explanation: B. In Germany, It is called as constructive motion of no-confidence, which is followed in Germany. It contains two motions- the first motion expresses lack of confidence of the house in the council of ministers and the second motion expresses confidence in the leader from opposition.

4. Which one of the following was NOT proposed by the 73rd Constitutional Amendment in the area of Panchayati Raj?

Explanation: C. Under Article 243D, at least one-third of the seats have been reserved for women in Panchayats. State Finance Commission shall be constituted to review the financial position of the panchayats under Article 243I.

5. Which one of the following States of India has passed a legislation (in 1996) making the maintenance of one's parents mandatory?

Explanation: D. Himachal Pradesh passed a legislation (in 1996) making the maintenance of one

6. Proportional representation is NOT necessary in a country where

Explanation: B. In proportional representation, a candidate requires 50%+ 1 vote to get elected. In a two party system, one automatically secures it. Hence proportional representation is not necessary in such country.

7. State Funding of elections takes place in

Explanation: D. In Germany, U.K, Denmark, Finland, Italy, Norway, Austria, USA, and Netherlands, state funding of elections is in practice.

8. The concept of Public Interest Litigation originated in

Explanation: C. The concept of Public Interest Litigation (PIL) originated in the United States. PIL is a legal action initiated in a court of law for the enforcement of public Interest or general interest. In India, the PIL is a product of the judicial Activist role of supreme Court. It was introduced in early 1980s. Justice V.R.Krishna Iyer and P.N. Bhagawati were the pioneers of the concept of PIL in India.

9. If the Prime Minister of India belonged to the Upper House of Parliament

Explanation: A. The Prime Minister of India belonging to Upper House will not be able to vote in his favour in the case of a no-confidence motion because no-confidence is passed in Lok Sabha only as per the Rule198 of the Lok Sabha. The PM can make his statements in either of the house of parliament. The Indian Constitution does not require the PM to be necessarily a member of Lok Sabha. However Article 75 says that Council of Ministers shall be collectively responsible to Lok Sabha.

10. Consider the following statements:
  1. Assertion (A): The reservation of thirty three per cent of seats for women in Parliament and State Legislatures does not require Constitutional amendment.
  2. Reason (R): Political parties contesting elections can allocate thirty three per cent of seats they contest to women candidates without any Constitutional amendment.

In the context of the above two statements, which one of the following is correct ?

Explanation: D. The Reservation of 33% of seats for women in Parliament and State Legislature require Constitutional Amendment.

11. In the Presidential election in India, every elected member of the Legislative Assembly of a State shall have as many votes as there are multiples of one thousand in the quotient obtained by dividing the population of the State by the total number of the elected members of the Assembly. As at present (1997) the expression "population" here means the population as ascertained by the

Explanation: A. The expression Population means the population as ascertained at the last preceding census of which the relevant figures have been published. Here it can be interpreted as a reference to the 1991 Census, until 2026.

12. Which one of the following are/is stated in the Constitution of India?
  1. The President shall not be a member of either House of Parliament
  2. The Parliament shall consist of the President and two Houses

Choose the correct answer from the codes given below:

Explanation: B. Article 59 of the Constitution states that the President shall not be a member of the either House of the Legislature of any state. Article 79 says, there shall be a parliament for the Union which shall consist of the President and two Houses to be known respectively as the council of states and House of people.

13. Match List I with List II and select the correct answer using the codes given below the lists:

Explanation: B. President

14. In which one of the following countries are 75 per cent of seats in both Houses of Parliament filled on the basis of first-past the-post system and 25 per cent on the basis of Proportional Representation system of elections ?

Explanation: C. France

15. In the following quotation: "WE, THE PEOPLE OF INDIA, having solemnly resolved to constitute India into a Sovereign Socialist Secular Democratic Republic and to secure to all its citizens: JUSTICE, social, economic and political; LIBERTY of thought, expression, belief, faith and worship? EQUALITY of status and of opportunity? and to promote among them all? FRATERNITY assuring the dignity of the individual and the unity and integrity of the Nation? In our Constituent Assembly this 'X' .......... do hereby adopt, enact and give to ourselves this Constitution." 'X' stands for

Explanation: B. On 26th November 1949, the constitution received the signature of the President of the Constituent Assembly and was declared as passed.

16. Assertion (A): Willful disobedience or noncompliance of Court Orders and use of derogatory language about judicial behaviour amounts o Contempt of Court. Reason (R): Judicial activism cannot be practised without arming the judiciary with punitive powers to punish contemptuous behaviour.

In the context of the above two statements, which one of the following is correct ?

Explanation: B. Both A and B are true but R is not the correct explanation of A. Judicial Activism refers to court rulings that are partially or fully based on the Judges political or personal considerations, rather than existing laws.

17. Which one of the following Schedules of the Constitution of India contains provisions regarding Anti-Defection Act?

Explanation: D. Tenth Schedule contains provisions as to disqualification on ground of defection. It was introduced by 52nd Amendment Act, 1985. It provides for the disqualification of a member of a legislature if he defects from his party to some other party. The decision is taken by the presiding officer in this regard subject to the judicial review. Expectations to this provision include in case of merger when two-thirds of the members have agreed to such a merger. It also excludes a member who voluntarily gives up the membership of his part after being elected as presiding officer or rejoins it after he has left the office.

18. The Indian parliamentary system is different from the British parliamentary system in that India has

Explanation: D. Independence of Judiciary and Judicial review have been taken from the US Constitution. In the context of U.K, the parliament is supreme. The features of Indian Constitution contained in option A, B, C have been taken from British Constitution.

19. Which one of the following Schedules of the Constitution of India contains provisions regarding Anti-Defection Act?

Explanation: D. Tenth Schedule contains provisions as to disqualification on ground of defection. It was introduced by 52nd Amendment Act, 1985. It provides for the disqualification of a member of a legislature if he defects from his party to some other party. The decision is taken by the presiding officer in this regard subject to the judicial review. Expectations to this provision include in case of merger when two-thirds of the members have agreed to such a merger. It also excludes a member who voluntarily gives up the membership of his part after being elected as presiding officer or rejoins it after he has left the office.

20. The Indian parliamentary system is different from the British parliamentary system in that India has

Explanation: D. Independence of Judiciary and Judicial review have been taken from the US Constitution. In the context of U.K, the parliament is supreme. The features of Indian Constitution contained in option A, B, C have been taken from British Constitution.

21. Which one of the following statements about a Money Bill is not correct?

Explanation: A. A money bill can be introduced only in LS(not in RS) that too on the recommendation of the President.

22. The 73rd Constitution Amendment Act, 1992 refers to the

Explanation: C. 73rd Amendment act added Eleventh Schedule to the Constitution and Part IX, which provides for the Panchayati Raj System.

23. The Speaker can ask a member of the House to stop speaking and let another member speak. This phenomenon is known as

Explanation: D. Decorum means maintaining proper behavior. Interpellation means formal right of a Parliament to submit formal question to the government. Crossing the floor means to vote against the party lines. Yielding the floor means the speaker giving part of his or her speaking time to another speaker. While this practice in deliberative assemblies, unless specifically authorized in the rules.

24. Consider the following statements about the AttorneyGeneral of India:
  1. He is appointed by the President of India.
  2. He must have the same qualifications as are required for a Judge of the Supreme Court.
  3. He must be a member of either House of Parliament.
  4. He can be removed by impeachment by Parliament.

Which of these statements are correct?

Explanation: A. Under Article 76, impeachment procedure of the Attorney-General is not provided. He shall hold office during the pleasure of the President. He must not be a member of either House of Parliament.

25. The primary function of the Finance Commission in India is to

Explanation: A. According to Article 280 it shall be the duty of the Commission to make recommendations to the President as to (1) the distribution between the Union and the States of the net proceeds of taxes which are to be, or may be, divided between them under this Chapter and the allocation between the states of the respective shares of such proceeds; (2) the principles which should govern the grants in aid of the revenues of the states out of Consolidated Fund of India; (3) any other matter referred to the commission shall determine their procedure and shall have such powers in the performance of their functions as parliament may by law confer on them.

26. The Parliament can make any law for the whole or any part of India for implementing International treaties

Explanation: D. It is the sole prerogative of Parliament under Article 253 of the Constitution.

27. The State which has the largest number of seats reserved for the Scheduled Tribes in Lok Sabha is

Explanation: D. Madhya Pradesh is the state, where the largest number of seats are reserved for scheduled tribe in the Lok Sabha.

28. Consider the following functionaries:
  1. Cabinet Secretary
  2. Chief Election Commissioner
  3. Union Cabinet Ministers
  4. Chief Justice of India

Their correct sequence, in the Order of Precedence is

Explanation: C. Chief Justice of India, Union Cabinet Minister, Chief Election Commissioner and Cabinet Secretary is the correct order of Precedence.

29. A college student desires to get elected to the Municipal Council of his city. The validity of his nomination would depend on the important condition, among others, that

Explanation: D. Provision given in option D is not included under 73rd Amendment Act 1993.

30. Match List I with List II and select the correct answer using the codes given below the Lists:
  1. A. Zila Parishads at the sub-divisional level 1. Andhra Pradesh
  2. B. Mandal Praja Parishad 2. Assam
  3. C. Tribal Councils 3. Mizoram
  4. D. Absence of Village Panchayat 4. Meghalaya

Codes:

Explanation: D.

31. The following news item appeared in a National daily dated 1 12-1999: "...Parliament today rejected a Bill to grant women the right to vote and stand for office in parliamentary elections, by a margin of 32 to 30. The National Assembly was split between liberal, progovernment and Shiite Muslim deputies who were in favour of women's rights, while the opposition camp grouped Sunni Muslim fundamentalists and tribal MPs. A total of 64 MPs and Ministers were present, of whom two abstained." The Parliament referred to in this quotation is that of

Explanation: B. Kuwait's all-male parliament rejected granting women full political rights. The Vote was 32 to 30 with 2 abstentions. With 64 members present, the bill needed 33 votes to pass.

32. Which Article of the Constitution provides that it shall be the endeavour of every state to provide adequate facility for instruction in the mother tongue at the primary stage of education?

Explanation: C. Article 350A was inserted by 7th Constitutional Amendment act 1956. It says, it shall be the endeavour of every State and of every local authority within the State to provide adequate facilities for instruction in the mother-tongue at the primary stage of education to children belonging to linguistic minority groups; and the President may issue such directions to any State as he considers necessary or proper for securing the provision of such facilities. Article 349

33. Which one of the following statements correctly describes the Fourth Schedule of the Constitution of India?

Explanation: D. Fourth schedule allocates seats in the Council of States i.e. Rajya Sabha.

34. Match List-I with List-II and select the correct answer using the codes given below the lists:

Explanation: A. A-5; B-1; C-4; D-2

35. If a new state of the Indian Union is to be created, which one of the following schedules of the Constitution must be amended?

Explanation: A. First schedule contains names of the States and UTs, that's why it should be amended, if a new state is created.

36. Match List-I with List-II and select the correct answer using the codes given below the lists:

Explanation: A. A-1; B-2; C-3; D-4

37. Which one of the following amendments to the Indian Constitution empowers the President to send back any matter for reconsideration by the Council of Ministers?

Explanation: D. 44th Before the 42nd amendment, Article 74(1) stated that, "there shall be a Council of Ministers with the Prime Minister at the head to aid and advise the President in the exercise of his functions". However, there was a slight ambiguity whether the advice of the Council of Ministers is binding on the President. Forty-second Amendment of the 42nd Constitutional Amendment (1976) made it explicit that the President shall, "act in accordance with such advice". The amendment went into effect from 3 January, 1977. The 44th Amendment (1978) however added that the President can send the advice back for reconsideration once. But if the Council of Ministers sends the same advice again to the President then the President must accept it. The amendment went into effect from 20 June, 1979.

38. The 93rd Constitution Amendment deals with the:

Explanation: B. According to 93rd Amendment every Child of the age group of 6-14 years shall have right to free and compulsory Education. No child is liable to pay any kind of fee/ capitation fee/ charges. A collection of capitation fee invites a fine up to 10 times the amount collected.

39. Match List-I (Article of Indian Constitution) with List -II (Provisions) and select the correct answer using the codes given below the lists:
  1. 1. Article 16 A. No person shall be deprived of his property save by the authority of law
  2. 2. Article 29 B. No person can be discriminated against in the matter of public appointment on the ground of race, religious or caste
  3. 3. Article 30 C. All minorities whether based on religion or language shall have to establish and administer educational institutions of their choice
  4. 4. Article 31 D. No citizen shall be denied admission into any educational institution maintained by the State, or receiving State aid, on grounds of religion, race, caste, language or any of them

Codes:

Explanation: A. These are Fundamental Rights under Part III of the Constitutionm, runs from Art 14

40. The purpose of the inclusion of Directive Principles of the State Policy in the Indian Constitution is to establish:

Explanation: D. The purpose of Directive Principles of State Policy is to establish the social and economic democracy. Political democracy is established by the Fundamental Rights.

41. Which one of the following Articles of the Directive Principles of the State Policy deals with the promotion of international peace and security?

Explanation: A. This comes under Directive Principles of State Policy under part IV of the Constitution. Article 51 states that the state shall endeavour to promote international peace and security.

42. Which one of the following statements is correct?

Explanation: C. In Rajya Sabha, 12 members are nominated by the President from the persons who have special knowledge in art, science, literature and social service. In Lok Sabha, 2 members are nominated by the President from the Anglo-Indian community (Art 331). A nominated member can vote only in the Vice-Presidential elections.

43. Consider the following statements:
  1. The joint sitting of the two houses of the Parliament in India is sanctioned under Article 108 of the Constitution
  2. The first joint sitting of Lok Sabha and Rajya Sabha was held in the year 1961
  3. The second joint sitting of the two Houses of Indian Parliament was held to pass the Banking Service Commission (Repeal) Bill

Which of these statements is correct?

Explanation: D. So far three joint sittings have been held, First was held on Dowry Prohibition Bill, 1961, Second was held on Banking Service Commission (Repeal) Bill, 1978 and Third was held on Prevention of Terrorism Bill, 2002. The presiding officer of joint sitting is Speaker of the Lok Sabha.

44. Which one of the following Bills must be passed by each House of the Indian Parliamentary separately by special majority?

Explanation: D. According to Article 368 an amendment of this Constitution may be initiated only by the introduction of a Bill for the purpose in either House of Parliament, and when the Bill is passed in each House by a majority of the total membership of that House present and voting, it shall be presented to the President who shall give his assent to the Bill and thereupon the Constitution shall stand amended in accordance with the terms of the Bill.

45. As per Indian Protocol, who among the following ranks highest in the order of precedence?

Explanation: C. Order of precedence is President, Vice President, PM, Governor of State within their respective states, Former Presidents and Deputy PM, CJI and Speaker of LS.

46. Consider the following statements: The function(s) of the Finance commission is/are:
  1. to allow the withdrawal of the money out of the Consolidated Fund of India
  2. to allocate between the States the shares of proceeds of taxes
  3. to consider applications for grants-in-aid from States
  4. to supervise and report on whether the Union and State governments are levying taxes in accordance with the budgetary provisions

Which of these statements is/are correct?

Explanation: B. Article 280 of the Constitution of India provides for a Finance Commission as a quasi-judicial body. It is constituted by the President of India every fifth year or at such earlier time as he considers necessary. The commission makes recommendations to the president with regard to the distribution of the proceeds of taxes between the union and the states. The principles which should govern the grants-in-aid to be given to the states. Any other matter referred to the Commission by the President in the interest of sound finance.

47. Consider the following statements: In the electoral college for Presidential Election in India,
  1. the value of the vote of an elected Member of Legislative Assembly equals State Population/ Number of Elected MLAs of the State
  2. the value of the vote of an elected Member of Parliament equals to total value of the votes of all elected MLA's and total number of elected MP's
  3. there were more than 5000 members in the latest elections.

Which of these statements is/are correct?

Explanation: B. the value of the vote of an elected Member of Parliament equals to total value of the votes of all elected MLA's and total number of elected MP's.

48. Consider the following statements:
  1. While members of the Rajya Sabha are associated with Committees on Public Accounts and Public Undertakings, members of Committee on Estimates are drawn entirely from the Lok Sabha
  2. The Ministry of Parliamentary Affairs works under the overall direction of Cabinet Committee on Parliamentary Affairs
  3. The Ministry of Parliamentary Affairs nominates Members of Parliament on Committees, Councils, Board and Commissions etc. set up by the Government of India in the various ministries.

Which of these statements is are correct?

Explanation: A. The PAC is formed every year with a strength of not more than 22 members of which 15 are from Lok Sabha, the lower house of the Parliament, and 7 from Rajya Sabha, the upper house of the Parliament. The term of office of the members is one year. The Estimates Committee, constituted for the first time in 1950, is a Parliamentary Committee consisting of 30 Members, elected every year by the Lok Sabha from amongst its Members. The Ministry of Parliamentary Affairs renders secretarial assistance to the Cabinet Committee on Parliamentary Affairs, which, interalia recommends prorogation of both the Houses of the Parliament, Govt's stand on Private Members' Bills and Resolutions. Option 3 is not correct.

49. Survey of India is under the ministry of:

Explanation: D. Survey of India, The National Survey and Mapping Organization of the country under the Department of Science and Technology, is the oldest scientific department of the Govt. of India. It was set up in 1767.

50. Which one of the following statements is NOT correct?

Explanation: C. Madhya Pradesh publishes largest number of newspapers in India.

51. The power to enlarge the jurisdiction of the Supreme Court of India with respect to any matter included in the Union List of Legislative Powers rests with:

Explanation: C. The Parliament

52. Which one of the following High Courts has the Territorial Jurisdiction over Andaman and Nicobar Islands?

Explanation: B. The U.T. of Andaman and Nicobar Islands comes under the jurisdiction of high court of Calcutta under its extended jurisdiction Act 1953.

53. Which one of the following Articles of the Constitution of India says that the executive power of every State shall be so exercised as not to impede or prejudice the exercise of the executive power of the Union?

Explanation: A. Article 257 in the Constitution states that the executive power of every State shall be so exercised as not to impede or prejudice the exercise of the executive power of the Union, and the executive power of the Union shall extend to the giving of such directions to a State as may appear to the Government of India to be necessary for that purpose. Article 258: Power of the Union to confer powers on the States in certain cases; Article 355: Duty of the Union to protect States against external aggression and internal disturbance; Article 358: Suspension of provisions of Article 19 during emergencies.

54. Match (Articles of the Constitution of India) with (Provision) and select the correct answer using the codes given below the lists:
  1. A. Article 14 1. The State shall not discriminate against any citizen on grounds only of religion, race, caste, sex place of birth or any of term
  2. B. Article 15 2. The State shall not deny to any person equality before the law or the equal protection of laws within the territory of India
  3. C. Article 16 3. Untouchability' is abolished and its practice in any from is forbidden
  4. D. Article 17 4. There shall be equality of opportunity for all citizens in matters relating to employment or appointment to any office under the State

Codes:

Explanation: C. These are Fundamental rights under Part III of the Constitution.

55. Which Article of the Constitution of India says, 'No child below the age of fourteen years shall the employed to work in any factory or mine or engaged in any other hazardous employment'?

Explanation: A. Article 24 of the constitution states that, no child below the age of fourteen years shall be employed to work in any factory or mine or engaged in any other hazardous employment.

56. With reference to Indian Parliament, which one of the following is not correct?

Explanation: A. Appropriation Bill is a money bill. In case of money bill, RS has only recommendatory power and need not to be passed by RS.

57. Consider the following tasks:
  1. Superintendence, direction and conduct of free and fair elections
  2. Preparation of electoral rolls for all elections to the Parliament, state Legislatures and the Office of the President and the Vice-President
  3. Giving recognition to political, parties and allotting election symbols to political parties and individuals contesting the election.
  4. Proclamation of final verdict in case of election disputes

Which of the above are the functions of the Election Commission of India?

Explanation: A. The High Court (and not the Election Commission) is the final authority to give a final verdict in case of election disputes. In the alternative special election benches may be constituted in high courts and earmarked exclusively for the disposal of election petitions and disputes.

58. Consider the following statements:
  1. The Speaker of Lok Sabha has the power to adjourn the House sine die but, on prorogation, it is only the President who can summon the House
  2. Unless sooner dissolved or there is an extension of the term, there is an automatic dissolution of the Lok Sabha by efflux of time, at the end of the period of five years, even if no formal order of dissolution is issued by the President
  3. The Speaker of Lok Sabha continues in office even after the dissolution of the House and until immediately before the first meeting of the House

Which of the statements given above are correct?

Explanation: D. Given all statements are correct.

59. Which one of the following statements is not correct?

Explanation: A. In case of a No-confidence motion, there is no need to set out the grounds on which it is based. No- Confidence motion is introduced only in the Lok Sabha by the opposition and needs a support of not less than 50 members of LS for its introduction. Rule 198 of the Lok Sabha specifies the procedure for a motion of noconfidence. Any member may give a written notice; the speaker shall read the motion of no confidence in the House and ask all those persons to rise who favours that the motion be taken up. If there are 50 MPs in favour, the speaker allots a date for discussing the motion.

60. The resolution for removing the Vice-President of India can be moved in the:

Explanation: D. Article 67(b) in the Constitution of India states, a Vice President may be removed from his office by a resolution of the council of States passed by a majority of all the then members of the council and agreed to by the House of the People; but no resolution for the purpose of this clause shall be moved unless at least fourteen days notice has been given of the intention to move the resolution.

61. With reference to the Constitution of India, which one of the following pairs is not correctly matched?

Explanation: B. Stock Exchanges are listed in the Seventh Schedule (Article 246) List I-Union List, item no. 90 that reads, taxes other than stamp duties on transactions in stock exchanges and futures markets. Forests Concurrent List, 17-A, Post Office Savings Bank -Union List-3, Public health and sanitation; hospitals and dispensaries - State List -6.

62. Consider the following statements:
  1. The highest deciding body for planning in India is the Planning Commission of India
  2. The Secretary of the Planning Commission of India is also the Secretary of National Development Council
  3. The Constitution includes economic and social planning in the Concurrent List in the Seventh Schedule of the Constitution of India

Which of the statements given above is/are correct?

Explanation: B. The highest decision making body for planning in India is the Parliament, National Development Council is at second position and Planning Commission at third position. Economic and social planning is placed under Entry 20 in the Concurrent list.

63. Consider the following events:
  1. Fourth general elections in India
  2. Formation of Haryana state
  3. Mysore named as Karnataka state
  4. Meghalaya and Tripura become full states

Which one of the following is the correct chronological order of the above?

Explanation: A. Haryana was founded in 1966 when the former state of Punjab was divided into Haryana and the modern Punjab. The 1967 general elections or the 4th Lok Sabha was held in February. Tripura, Manipur and Meghalaya became full-fledged states on Jan 21, 1972. Mysore state was renamed Karnataka in 1997.

64. With reference to Indian public finance, consider the following statements:
  1. Disbursements from Public Accounts of India are subject to the Vote of the Parliament
  2. The Indian Constitution provides for the establishment of a Consolidated Fund, a Public Account and a Contingency Fund for each State
  3. Appropriations and disbursements under the Railway Budget are subject to the same form of parliamentary control as other appropriations and disbursements

Which of the statements given above are correct?

Explanation: B. The Indian Constitution provides for the establishment of a Consolidated Fund and a Public Account under Article 266 and a Contingency Fund for each State under Article 267. Disbursements from Public Accounts of India are not subject to a Vote of Parliament.

65. The Archaeological Survey of India is an attached office of the Department/Ministry of:

Explanation: A. The Archaeological Survey of India, established in 1861 is a department of the Government of India attached to the Ministry of Culture. ASI is responsible for archaeological studies and the preservation of archaeological heritage of the country in accordance with the various acts of the Indian Parliament.

66. Assertion (A): The Central Rural Sanitation Programme was launched in 1986 to improve the quality of life of rural people in India. Reason (R): Rural sanitation is a subject in the Concurrent List in the Constitution of India. In the context of above two statements, which one of the following is correct?

Explanation: C. Rural sanitation is not a subject in the Concurrent List. Public Health and Sanitation comes under the State List. Personal and food hygiene have been major cause of many diseases in developing countries. It was in this context that the central Rural Sanitation Programmer (CRSP) was launched in 1986.

67. Consider the following statements:
  1. The highest criminal court of the district is the Court of District and Session Judge
  2. The District Judge are appointed by the Governor in consultation with the High Courts.
  3. A person to be eligible for appointment as a District Judge should be an advocate or a pleader of seven years' standing or more, or an officer in judicial service of the Union or the State.
  4. When the sessions judge awards a death sentence, it must be confirmed by the High Court before it is carried out.

Which of the statements given above are correct?

Explanation: D. These provisions are given under Article 233-235 in the chapter of Subordinate Courts in the Constitution of India.

68. According to the National Human Rights Commission Act, 1993, who amongst the following can be its Chairman?

Explanation: C. According to NHRC Act 1993, only a retired CJI can become chairman of NHRC, appointed by President on the recommendation of a committee comprising of PM, Speaker of Lok Sabha, Home Minister, Leader of Opposition of both Houses of Parliament and Deputy Chairman of Rajya Sabha

69. Which one of the following is the correct sequence in the descending order of precedence in the warrant of precedence?

Explanation: B. President comes first, Vice-President second, Prime Minister third and Governors of states with in their respective State comes fourth in the Warrant of Precedence. According to Indian order of precedence, Judges of the Supreme Court

70. Consider the following statements:
  1. Part IX of the Constitution of India provisions for Panchyats and was inserted by the Constitution (Amendment) Act, 1992.
  2. Part IX A of the Constitution of India contains provisions for Municipalities and the Article 243 Q envisages two types of Municipalities a Municipal Council and a Municipal Corporation for every State.

Which of the statements given above is/are correct?

Explanation: A. Option A is correct.

71. Consider the following statements:
  1. Article 371 A to 371 I were inserted in the Constitution of India to meet regional demands of Nagaland, Assam, Manipur, Andhra Pradesh, Sikkim, Mizoram, Arunchal Pradesh and Goa.
  2. Constitution of India and the United States of America envisage a dual policy (The Union and the States) but a single citizenship.
  3. A naturalized citizen of India can never be deprived of his citizenship.

Which of the statements given above is/are correct?

Explanation: D. Article 371 A to I deals with special provisions to -Nagaland, Assam, Manipur, Andhra Pradesh, Sikkim, Mizoram, Arunachal Pradesh and Goa. The Constitution of India envisages a single policy for both Union and the States. A naturalized citizen is one who acquires citizenship either by Naturalization or by Registration. They can be deprived of citizenship if they acquired citizenship by using fraudulent means.

72. The Constitution (98th Amendment) Act is related to:

Explanation: B. The Constitution 98th Amendment Bill, 2003, seeks to constitute a National Judicial Commission (NJC) by including Chapter IV-A in Part V of the Constitution which will be in charge of appointing judges to the higher judiciary and for transferring High Court Judges.

73. Consider the following statements:
  1. The Constitution of India has 20 parts.
  2. There are 390 Articles in the Constitution of India in all.
  3. Ninth, Tenth, Eleventh and Twelfth Schedules were added to the Constitution of India by the Constitution (Amendment) Acts.

Which of the statements given above is/are correct?

Explanation: C. The Constitution of India has 24 parts, 12 schedules and more than 444 articles at present. In the original constitution, there were 22 parts, 8 schedules and 395 articles. Ninth Schedule was added by 1st Constitutional Amendment Act, 1951. Tenth Schedule was added by 52nd Constitutional Amendment Act, 1985. Eleventh Schedule was added by 73rd Constitutional Amendment Act, 1992. Twelfth Schedule was added by 74th Constitutional Amendment Act, 1992.

74. Who among the following was the chairman of the Union Constitution Committee of the Constituent Assembly?

Explanation: C. Chairman of different Committees of Constituent Assembly: Union Power Committee

75. Under which one of the Ministries of the Government of India does the Food and Nutrition Board work?

Explanation: C. Food and Nutrition Board works under Ministry of Women and Child Development. It is a technical support wing under Child Development Bureau of the Ministry. None of the given options is correct.

76. Consider the following:
  1. Disputes with mobile cellular companies
  2. Motor accident cases
  3. Pension cases

For which of the above are Lok Adalats held?

Explanation: D. Option D is correct.

77. Consider the following statements:
  1. There are 25 High Courts in India.
  2. Punjab, Haryana and the Union Territory of Chandigarh have a common High Court.
  3. National Capital Territory of Delhi has a High Court of its own.

Which of the statements given above is/are correct?

Explanation: A. There were 21 High Courts in India with three new states created in 2000, having their own High Courts (Chattisgarh at Bilaspur, Uttarakhand at Nainital and Jharkhand at Ranchi). Punjab, Haryana and Chandigarh have a common HC at Chandigarh. In the year 2013, three new High Courts in the northeast - Meghalaya, Manipur and Tripura were created taking the total number of High Courts in the country from 21 to 24. National Capital Territory of Delhi has a High Court of its own which was established in the year 1966. As if 2018. There are 24 High courts in India.

78. Consider the following statements:
  1. The Parliament cannot enlarge the jurisdiction of the Supreme Court of India as its jurisdiction is limited to that conferred by the Constitution.
  2. The officers and servants of the Supreme Court and High Courts are appointed by the concerned Chief Justice and the administrative expenses are charged on the Consolidated fund of India.

Which of the statements given above is/are correct?

Explanation: B. Statement (1) is not correct as according to Article 138(1) of the Constitution, Parliament can enlarge the jurisdiction and powers of the SC w.r.t. to any of the matters in the Union List. Whereas SC

79. Consider the following statements:
  1. Article 301 pertains to the Right to Property.
  2. Right to Property is a legal right but not a Fundamental Right.
  3. Article 300 A was inserted in the Constitutional Amendment

Which of the statement given above is/are correct?

Explanation: A. Article 301 pertains to Freedom of Trade, Commerce and Intercourse. In the original constitution right to property was a Fundamental Right under Article 19(1) (f). But 44th Amendment Act, 1978 omitted sub clause f, and inserted Article 300A to make right to property a legal right.

80. What does the 104th Constitution Amendment Bill related to?

Explanation: C. The Indian Parliament has passed the 104th Constitution Amendment Bill, providing reservations for the socially and educationally backward classes, besides the Scheduled Classes and Scheduled Tribes, in all private aided and unaided educational institutions.

81. Which one among the following commission was set up in pursuance of a definite provision under an Article of the Constitution of India?

Explanation: C. Superintendence, direction and control of elections to be vested in an Election Commission under Article 324.

82. Consider the following statements:
  1. Free and compulsory education to the children of 6-14 years age-group by the State by the seventysixth Amendment to the Constitution of India.
  2. Sarva Shiksha Abhiyan seeks to provide computer education even in rural areas.
  3. Education was included in the Concurrent List by the Forty-second Amendment, 1976 to the 'Constitution of India'.

Which of the statements given above are correct?

Explanation: C. Statement 1 is incorrect as this provision was added by 86th Amendment Act (not 76th).

83. Consider the following statements:
  1. The Rajya Sabha alone has the power to declare that it would be in national interest for the Parliament to legislate with respect to a matter in the State List.
  2. Resolutions approving the proclamation of Emergency are passed only by the Lok Sabha.

Which of the statement(s) given above is/are correct?

Explanation: A. Statement 1 is correct as per provisions under Article 249. Statement 2 is incorrect as resolutions approving the proclamation of Emergency are passed by both Houses of Parliament (not only LS).

84. Consider the following statements:
  1. There is no provision in the Constitution of India to encourage equal pay for equal work for both men and women.
  2. The Constitution of India does not define backward classes.

Which of the statement(s) given above is/are correct?

Explanation: B. Statement 1 is incorrect as equal pay for equal work for both men and women is provided under Article 39(d) in Directive Principles of the State Policy under Part IV of the Constitution. Under Article

85. Assertion (A): In India, every State has a High Court in its territory. Reason (R): The Constitution of India provides a High Court in each State.

Explanation: D. Article 214 of the Constitution says

86. Who was the Chief Justice of India when Public Interest Litigation (PIL) was introduced to the Indian judicial system?

Explanation: D. PN Bhagwati was CJI during July 1985

87. Consider the following statements:
  1. A person who has held office as a permanent Judge of a High Court cannot plead or act in any court or before any authority in India except of the Supreme Court.
  2. A person is not qualified for appointment as a Judge of a High Court in India unless he has for at least five years held a judicial office in the territory of India.

Which of the statement(s) given above is/are correct?

Explanation: D. Statement 1 is incorrect because after retirement a permanent judge of High Court shall not plead or act in a Court or before any authority in India, except the SC and a HC other than the HC in which he had held his office (Art 220).

88. Which of the following Constitution Amendment Acts seeks that the size of the Councils of Ministers at the Centre and in a State must not exceed 15 per cent of the total number of members in the Lok Sabha and the total number of members of the Legislative Assembly of that State, respectively?

Explanation: A. The above provision has been added by 91st constitutional Amendment Act, 2003.

89. Assertion (A): The Council of Ministers in the Union of India is collectively responsible both to the Lok Sabha and Rajya Sabha. Reason (R): The Members of both the Lok Sabha and the Rajya Sabha are eligible to be the Ministers of the Union Government.

Explanation: D. Assertion is false, because the Council of Ministers in the Union of India is collectively responsible to the Lok Sabha only.

90. Consider the following statements:
  1. Jawaharlal Nehru was in his fourth term as the Prime Minister of India at the time of his death.
  2. Jawaharlal Nehru represented Rae Bareilly constituency as a Member of Parliament.
  3. The first non-Congress Prime Minister of India assumed the office in the year 1977.

Which of the statements given above is/are correct?

Explanation: D. Terms of Jawahar Lal Nehru (1947-52; 1952-57; 1957-62; 1962-64). He died in 1964 during his 4th Prime-ministerial term. He represented Phulpur constituency in UP. The first non-congress PM of India was Morarji Desai (Janta Party) from 1977-1979.

91. Who was the Speaker of the First Lok Sabha?

Explanation: B. GV Mavalankar (1952-56), Hukum Singh (1962-67); K.M. Munshi and U.N. Dhebar were never the Speakers of the Lok Sabha.

92. Consider the following statements in respect of financial emergency under Article 360 of the Constitution of India:
  1. A proclamation of financial emergency issued shall cease to operate at the expiration of two months, unless before the expiration of that period it has been approved by the resolutions of both Houses of Parliament.
  2. If any proclamation of financial emergency is in operation, it is competent for the President of India to issue directions for the reduction of salaries and allowances of all or any class of persons serving in connection with the affairs of the Union but excluding the Judges of Supreme Court and the High Courts.

Which of the statements given above is/are correct?

Explanation: A. Statement 1 is correct as under Article 360, any Proclamation of Financial Emergency issued shall cease to operate at the expiration of two months, unless before the expiration of that period it has been approved by the resolutions Passed by both Houses of Parliament. If approved by both Houses, then it operates for 6 months. During the proclamation of financial emergency. President of India can issue directions for the reduction of salaries and allowances of all or any class of persons serving in connection with the affairs of the union including the judges of supreme court and the High warts.

93. Consider the following statements:
  1. The Chairman of the Committee on Public Accounts is appointed by the Speaker of the Lok Sabha.
  2. The Committee on Public Accounts comprises Members of Lok Sabha, Members of Rajya Sabha and few eminent persons of industry and trade.

Which of the statements given above is/are correct?

Explanation: A. Statement 2 is incorrect as Public Accounts Committee consists of 22 members: 15 from Lok Sabha and 7 from Rajya Sabha.

94. Consider the following statements:
  1. The nation-wide scheme of the National Child Labour Projects (NCLP) is run by the Union Ministry of Social Justice and Empowerment.
  2. Gurupadswamy Committee dealt with the issues of child labour.

Which of the statements given above is/are correct?

Explanation: B. Correct option is B.

95. Consider the following statements:
  1. The mode of removal of a Judge of a High Court in India is same as that of removal of a Judge of the Supreme Court.
  2. After retirement from the office, a permanent judge of a High Court cannot plead or act in any court or before any authority in India.

Which of the statements given above is/are correct?

Explanation: A. Statement 2 is incorrect. After retirement a permanent judge of High Court shall not plead or act in a Court or before any authority in India, except the SC and a HC other than the HC in which he had held his office (Art 220).

96. Consider the following statements:
  1. Justice V R Krishna Iyer was the Chief Justice of India.
  2. Justice V R Krishna Iyer is considered as one of the progenitors of Public Interest Litigation (PIL) in the Indian judicial system.

Which of the statements given above is/are correct?

Explanation: B. Justice P. N. Bhagwati and Justice V. R. Krishna Iyer were among the first judges to admit PIL's in the court.

97. Who among the following have held the office of the Vice-President of India?
  1. Mohammad Hidayatullah
  2. Fakhruddin Ali Ahmed
  3. Neelam Sanjiva Reddy
  4. Shankar Dayal Sharma

Select the correct answer using the code given below:

Explanation: B. Mohd. Hidayatullah (1979-84); Shankar Dayal Sharma (1987-92).

98. Match List-I with List-II and select the correct answer using the code given below the Lists:
  1. A. Nagendra Singh 1. Chief Election Commissioner of India
  2. B. A. N. Ray 2. President, International Court of Justice
  3. C. R. K. Trivedi 3. Chief Justice of India
  4. D. Ashok Desai 4. Attorney General of India

Code:

Explanation: B. Ashok Desai was Attorney General of India (1996-98); Nagendra Singh was President of International Court of Justice (1985-88); AN Ray was CJI (1973-77); RK Trivedi was Chief Election Commissioner (1982-85).

99. Consider the following statements: The Constitution of India provides that:
  1. the Legislative Assembly of each State shall consist of not more than 450 members chosen by direct election from territorial constituencies in the State
  2. a person shall not be qualified to be chosen to fill a seat in the Legislative Assembly of a State if he/ she is less than 25 years of age

Which of the statements given above is/are correct?

Explanation: B. Statement 2 is correct as per provisions given under Article 173. Statement 1 is incorrect as according to article 170, the legislative assembly of each state shall consist of not more than 500 and not less than 60 members chosen by direct election from territorial constituencies in the state.

100. Which of the following is/are included in the Directive Principles of the State Policy?
  1. Prohibition of traffic in human beings and forced labour
  2. Prohibition of consumption except for medicinal purposes of intoxicating drinks and of other drugs which are injurious to health

Select the correct answer using the code given below:

Explanation: B. Statement 1 is incorrect as it is a Fundamental Right under article 23 of Part III of the constitution. Statement 2 corresponds to Directive Principles of State Policy under Article 47 under Part IV of the Constitution.

101. Which Schedule of the Constitution of India contains: Special provisions for the administration and control of Scheduled Areas in several States?\

Explanation: B. Fifth schedule relates to the control and administration of scheduled areas in states other than Tripura, Assam, Meghalaya and Mizoram. While Sixth Schedule deals with administration and control of tribal areas in the state of Assam, Meghalaya, Mizoram and Tripura.

102. Under which one of the following Constitution Amendment Acts, four languages were added to the list of languages under the Eighth Schedule of the Constitution of India, thereby raising their number to 22?

Explanation: C. 92nd Amendment Act 2003 added Bodo, Santhali, Maithali and Dogri languages in the 8th Schedule of the Constitution. Originally there were 14 languages in the 8th schedule. 21st amendment act added Sindhi language. 71st amendment act added Konkani, Manipuri and Nepali languages.

103. Which one of the following Constitutional Amendments states that the total number of Ministers, including the Prime Minister, in the Council of Minister shall not exceed fifteen percent of the total number of members of the House of the People?

Explanation: B. The above provision has been added by 91st Constitutional Amendment Act 2003.

104. With reference to the Union Government, consider the following statements:
  1. The Constitution of India provides that all Cabinet Ministers shall be compulsorily be a sitting members of Lok Sabha only.
  2. The Union Cabinet Secretariat operates under the direction of the Ministry of Parliamentary Affairs.

Which of the statements given above is/are correct?

Explanation: D. Statement 1 is incorrect as members of Rajya Sabha can become cabinet ministers. Statement 2 is incorrect as Cabinet secretariat is under the direct charge of the PM.

105. With reference to Union Government, consider the following statements:
  1. The Ministries and Departments of the Government of India are created by the Prime Minister on the advice of the Cabinet Secretary.
  2. Each of the ministries is assigned to a Minister by the President of India on the advice of the Prime Minister.

Which of the statements given above is/are correct?

Explanation: B. According to Article 70 the Prime Minister shall be appointed by the President and the other Ministers shall be appointed by the President on the advice of the Prime Minister. The government of India (Allocation of Business) Rules, 1961 is made by the President of India under Article 77 of the constitution for the allocation of business of the government of India. The Ministries/ Departments of the government of India are created by the president on the advice of the prime Minister under these Rules.

106. Consider the following statements:
  1. The Advocate General of a State in India is appointed by the President of India upon the recommendation of the Governor of the concerned State.
  2. As provided in Civil Procedure Code, High Courts have original, appellate and advisory jurisdiction at the State level.

Which of the statements given above is/are correct?

Explanation: B. Statement 1 is incorrect as Advocate General of the state is appointed by the governor of the State. Statement 2 is correct. The original civil jurisdiction of the High Courts has not altogether been abolished but retained in respect of actions of higher value. The Original Criminal Jurisdiction of High Courts has, however, been completely taken away by the criminal procedure code, 1973.

107. Consider the following statements:
  1. Central Administrative Tribunal (CAT) was set up during the Prime Ministership of Lal Bahadur Shastri.
  2. The Members of CAT are drawn from both judicial and administrative streams.

Which of the statements given above is/are correct?

Explanation: B. Statement 1 is incorrect as CAT was set up in1985 during the prime ministership of Rajiv Gandhi.

108. With reference to Union Government, consider the following statements:
  1. Number of Ministries at the Centre on 15th August 1947 was 18.
  2. Number of Ministries at the Centre at present is 36.

Which of the statements given above is/are correct?

Explanation: A. The number of ministeries at the Centre can vary based on factors such as volume of work, importance attached to different sectors, changes of orientation of policy etc. On 15 Aug, 1947, the number of ministries at the centre was 18.

109. If a Panchayat is dissolved, elections are to be held within:

Explanation: C. Duration of Panchayats is five year. Fresh election to constitute a Panchayat shall be completed before the expiry of its term; or in case of dissolution fresh election is to be conducted before the expiry of a period of 6 months from the date of its dissolution.

110. In India, the first Municipal Corporation was set up in which one among the following?

Explanation: B. In 1688, the first Municipal Corporation of India was set up in Madras.

111. With reference to Lok Adalats, consider the following statements:
  1. An award made by a Lok Adalat is deemed to be a decree of a civil court and no appeal lies against there to any court.
  2. Matrimonial/Family disputes are not covered under Lok Adalat.

Which of the statements given above is/are correct?

Explanation: A. Fundamental Rights available only to citizens and not to foreigners: Rights available under Article-15, 16, 19, 29 & 30. Fundamental Rights available to bath citizens and foreigners (except enemy aliens): Rights available under Article

112. Anekantavada is a core theory and philosophy of which one of the following?

Explanation: B. The Jaina metaphysics is realistic and relativistic pluralism. It is called Anekantavada or the doctrine of 'manyness of reality'. Matter and spirit are regarded as separate and independent realities.

113. Which one of the following amendments to the Indian Constitution empowers the President to send back any matter for reconsideration by the Council of Ministers?

Explanation: D. 44th Before the 42nd amendment, Article 74(1) stated that, "there shall be a Council of Ministers with the Prime Minister at the head to aid and advise the President in the exercise of his functions". However, there was a slight ambiguity whether the advice of the Council of Ministers is binding on the President. Forty-second Amendment of the 42nd Constitutional Amendment (1976) made it explicit that the President shall, "act in accordance with such advice". The amendment went into effect from 3 January, 1977. The 44th Amendment (1978) however added that the President can send the advice back for reconsideration once. But if the Council of Ministers sends the same advice again to the President then the President must accept it. The amendment went into effect from 20 June, 1979.

114. The 93rd Constitution Amendment deals with the:

Explanation: B. According to 93rd Amendment every Child of the age group of 6-14 years shall have right to free and compulsory Education. No child is liable to pay any kind of fee/ capitation fee/ charges. A collection of capitation fee invites a fine up to 10 times the amount collected.

115. Match List-I (Article of Indian Constitution) with List -II (Provisions) and select the correct answer using the codes given below the lists:
  1. A. Article 16 1. No person shall be deprived of his property save by the authority of law
  2. B. Article 29 2. No person can be discriminated against in the matter of public appointment on the ground of race, religious or caste
  3. C. Article 30(I) 3. All minorities whether based on religion or language shall have to establish and administer educational institutions of their choice
  4. D. Article 31 (I) 4. No citizen shall be denied admission into any educational institution maintained by the State, or receiving State aid, on grounds of religion, race, caste, language or any of them.

Codes:

Explanation: A. These are Fundamental Rights under Part III of the Constitution, runs from Art 14

116. With reference to Indian polity, which one of the following statements is correct?

Explanation: B. President can issue an ordinance only when both houses of parliament are not in session (Art 123). No minimum age is prescribed for appointment as a judge of the Supreme Court in the Constitution. The age of a Judge of the SC shall be determined by such authority and in such manner as parliament may by law provide. NDC is composed of the PM as its head, all Union Cabinet ministers, the CMs of all states, CMs/ Administrators of all UTs and the members of the Planning Commission. There is no constitutional provision regarding the accountability of the planning commission the parliament.

117. Consider the following statements with reference to India:
  1. The Chief Election Commission and other Election Commissioners enjoy equal powers but receive unequal salaries
  2. The Chief Election Commissioner is entitled to the same salary as is provided to a judge of the Supreme Court
  3. The Chief Election Commissioner shall not be removed from his office except in like manner and on like grounds as a judge of the Supreme Court
  4. The term of office of the Election Commissioner is five years from the date he assumes his office or till the day he attains the age of 62 years, whichever is earlier

Which of these statements are correct?

Explanation: B. The Chief Election Commissioner and other Election Commissioners enjoy equal powers and salaries. The term of office of the Election Commissioner is 6 years or till he attains the age of 65 years or whichever is earlier.

118. In the case of election to the Lok Sabha, the amount of Security deposited for general category candidates and SC/ST category candidates respectively is:

Explanation: C. As per the given options the right answer is (c). As if now (2018); The security deposit for Lok Sabah election is Rs 25,000 while for on Assembly election it is Rs 10,000. A de feasted indicate who fails to secure move than one sixth of the valid votes polled in the constituency will lose his security deposit.

119. The term of the Lok Sabha:

Explanation: C. The term of the LS can be extended by not more than one year at a time during the proclamation of national emergency under Article 352.

120. Which one of the following authorities recommends the principles governing the grants-in-aid of the revenues to the states out of the Consolidated Fund of India?

Explanation: A. The finance commission will be governed by the principles to allocate grants-in-aid of the revenues of the states out of the consolidated fund of India.

121. The Consultative Committee of Members of Parliament for Railway Zones is constituted by the:

Explanation: C. Main purpose of these Committees is to provide a forum for informal discussions between the Government and Members of Parliament on policies and programmes of the Government and the manner of their implementation.

122. The salaries and allowances of the Judges of the High Court are charged to the:

Explanation: B. The salaries and allowances of the Judges of the HC are charged to the Consolidated Fund of the state but their pensions are payable as Charged Expenditure.

123. In the Indian Constitution, the Right to Equality is granted by five Articles. They are:

Explanation: C. Fundamental Rights have been categorised into 6 groups: Right to Equality (14-18), Right to Freedom (19-22), Right against Exploitation (23 and 24), Right to Freedom of Religion (25 and 28), Cultural & Educational Rights (29 and 30), Right to Constitutional remedies (32).

124. Which one of the following rights was described by Dr. B.R. Ambedkar as the heart and soul of the Constitution?

Explanation: D. Right to Constitutional Remedies under article 32 is a Fundamental Right. It was called the very soul of Indian constitution and very heart of it, by B.R. Ambedkar.

125. With reference to the Constitution of India, consider the following:
  1. Fundamental Rights
  2. Fundamental Duties
  3. Directive Principles of the State Policy

Which of the above provisions of the Constitution of India is/are fulfilled by the National Social Assistance Programme launched by the government of India?

Explanation: B. The National Social Assistance Programme (NSAP) which came into effect from 15th August, 1995 represents a significant step towards the fulfillment of the Directive Principles in Article 41 (Right to Work, to Education and to Public Assistance in certain cases) of the Constitution.

126. In order to comply with TRIPS Agreement, India enacted the Geographical Indications of
  1. A Trade Mark is an individual or a company's right whereas a Geographical Indication is a community's right.
  2. A Trade Mark can be licensed whereas a Geographical Indication cannot be licensed.
  3. A Trade Mark is assigned to the manufactured goods whereas the Geographical-Indication is assigned to the agricultural goods/products and handicrafts only.

Which of the statements given above is/are correct?

Explanation: B. A trademark is a distinctive sign or indicator used by an individual, business organization, or other legal entity to identify that the products or services to consumers with which the trademark appears originate from a unique source, and to distinguish its products or services from those of other entities. The owner of a registered trademark may commence legal proceedings for trademark infringement to prevent unauthorized use of that trademark. However, registration is not required.

127. Which one of the following was not stipulated in the Fiscal Responsibility and Budget Management Act, 2003?

Explanation: C. It included recommendations for Fiscal deficit and revenue deficit.

128. With reference to the Consumer Disputes Redressal at district level in India, which one of the following statements is not correct?

Explanation: C. The District Forum entertains the complaints where the value of goods or services does not exceed rupees twenty lakhs.

129. With reference to Lok Adalats, which of the following statements is correct?

Explanation: D. Cases that are pending in regular courts can be transferred to a Lok Adalat if both the parties agree. These are usually presided over by retired judges, social Activists, or other members of the legal profession. Lok Adalats can deal with any matter falling within the jurisdiction whether it is of civil or criminal in nature.

130. The constitution (seventythird amendment) act, 1992, which aims at promoting the panchayati raj institutions in the country, provides for which of the following?
  1. Constitution of district planning committees.
  2. State election commissions to conduct all panchayat elections.
  3. Establishment of state finance commission.

State the correct answer using the codes given below:

Explanation: C. District planning committee has been provided by Article 243 ZD under Constitution of India, which was inserted by 74th Constitution Amendment Act and not by 73rd amendment act. So, first tricky statement was incorrect. Rest 2 statements are correct and correct option is C.

131. The authorization for the withdrawal of funds from the consolidated fund of India must come from ?

Explanation: B. Parliament shall have power to authorise by law the withdrawal of moneys from the Consolidated Fund of India for the purposes for which the said grants are made.

132. All revenues received by the union government by way of taxes and other receipts for the conduct of government business are credited to the

Explanation: C. Consolidated Fund of India.

133. When the annual budget is not passed by the lok sabha?

Explanation: D. If a policy statement like Annual Budget is not passed by the Lok Sabha, then it is equivalent to losing vote of confidence of the ruling party or it is same as passing of no-confidence motion. In this case, the government i.e. Prime Minister should resign.

134. Under the constitution of India, which one of the following is not a fundamental duty?

Explanation: A. Option A is not a fundamental duty.

135. With reference to the finance commission of India, which of the following statements is correct?

Explanation: D. The finance commission shall make recommendations as to the following matters namely: I. The distribution between the union and the states of the net proceeds of taxes which are to be, or may be, divided between them under Chapter I Part XII of the Constitution and the allocation between the states of the respective shares of such proceeds; II. The principles which should govern the garnts-in-aid of the revenues of the States out of the Consolidated Fund of India and the sums to be paid to the states which are in need of assistance by way of grants-in-aid of their revenues under Article 275 of the Constitution for purposes other than those specified in the provisions to clause (1) of that article; III. The measures needed to augment the consolidated Fund of a state to supplement the resources of the Panchayats and municipalities in the state on the basis of the recommendations made by the by the Finance Commission of the state.

136. Consider the following statements: In India, a metropolitan planning committee :
  1. Is constituted under the provisions of the constitution of India.
  2. Prepares the draft development plans for metropolitan area.
  3. Has the sole responsibility for implementing government sponsored schemes in the metropolitan area.

Which of the statements given above is/are correct?

Explanation: A. Statement 3 is not correct as it is not the sole authority for implementing government sponsored schemes in the metropolitan area. For example, in Delhi, there are various other agencies as well for implementing government sponsored schemes such as New Delhi Municipal Corporation, Delhi Development Authority.

137. What is the difference between "vote-on-account" and interim budget?
  1. The provision of a "vote-on-account" is used by a regular government, while an "interim budget" is a provision used by a caretaker government.
  2. A "vote-on-account" only deals with the expenditure in government is budget, while an "interim budget" includes both expenditure and receipts.

Which of the statements given above is/are correct?

Explanation: B. Every year the government presents the annual budget for the next financial year. However, if it is an election year the government would present a vote on account or interim budget. During an election year it is not practical for the ruling government or for the new government taking charge after the elections to prepare or debate on the full budget and pass it before the new financial year begins.

138. Consider the following:
  1. Right to education.
  2. Right to equal access to public service.
  3. Right to food.

Which of the above is/are Human Right/Human Rights under "Universal Declaration of Human Rights"?

Explanation: D. With reference to the Universal Declaration of Human Rights: Article 25 says everyone has the right to a standard of living adequate for the health and well-being of himself and of his family, including food, clothing, housing and medical care and necessary social services, and the right to security in the event of unemployment, sickness, disability, widowhood, old age or other lack of livelihood in circumstances beyond his control. Article 21(2) says

139. Consider the following provisions under the Directive Principles of State Policy as enshrined in the Constitution of India:
  1. Securing for citizens of India a uniform civil code
  2. Organizing village Panchayats
  3. Promoting cottage industries in rural areas
  4. Securing for all the workers reasonable leisure and cultural opportunities

Which of the above are the Gandhian Principles that are reflected in the Directive Principles of State Policy?

Explanation: B. Organizing village Panchayats and promoting cottage industries in rural areas are the Gandhian principles that are reflected in the Directive Principles of State Policy.

140. Which of the following special powers have been conferred on the Rajya Sabha by the Constitution of India?

Explanation: B. Rajyasabha can pass a resolution empowering the parliament to make laws in the state list and to create one or more All India Services. This is a special power that has been conferred on the Rajya Sabha by the constitution.

141. Which of the following provisions of the Constitution of India have a bearing on Education?
  1. Directive Principles of State Policy
  2. Rural and Urban Local Bodies
  3. Fifth Schedule
  4. Sixth Schedule

Select the correct answer using the codes given below :

Explanation: D. The items DPSP, Rural and Urban Local Bodies, 6th Schedule, 7th Schedule provide provision for education.

142. According to the Constitution of India, it is the duty of the President of India to cause to be laid before the Parliament which of the following?
  1. The Recommendations of the Union Finance Commission
  2. The Report of the Public Accounts Committee
  3. The Report of the Comptroller and Auditor General
  4. The Report of the National Commission for Scheduled Castes

Select the correct answer using the codes given below :

Explanation: C. CAG gives three audit reports to president which are laid by the president before both the houses of the parliament. Subsequently the Public Accounts committee examines them and reports its findings to the parliament.

143. Which of the following is/are among the Fundamental Duties of citizens laid down in the Indian Constitution?
  1. To preserve the rich heritage of our composite culture
  2. To protect the weaker sections from social injustice
  3. To develop the scientific temper and spirit of inquiry
  4. To strive towards excellence in all spheres of individual and collective activity

Select the correct answer using the codes given below:

Explanation: C. All the statements except 2 regarding the Fundamental Duties of citizens are correct.

144. The distribution of powers between the Centre and the States in the Indian Constitution is based on the Act provided in the

Explanation: C. Distribution of power between the Centre and the States in the Indian Constitution is based on the Government of India Act. 1935

145. In the Parliament of India, the purpose of an adjournment motion is

Explanation: A. Adjournment motion: It is introduced in the Parliament to draw attention of the house to a definite matter of urgent public importance. It is regarded as an extraordinary device, because it interrupts the normal business of the house. It involves an element of censure against the government and Rajya Sabha cannot introduce Adjournment Motion. The discussion on adjournment motion should last for not less than two hours and thirty minutes.

146. Consider the following statements:
  1. Union Territories are not represented in the Rajya Sabha.
  2. It is within the purview of the Chief Election Commissioner to adjudicate the election disputes.
  3. According to the Constitution of India, the Parliament consists of the Lok Sabha and the Rajya Sabha only.

Which of the statements given above is/are correct?

Explanation: D. None of the above statements are correct. Union Territories (Delhi and Pondicherry) are represented in the Rajya Sabha. It is not within the purview of the Chief Election Commissioner to adjudicate election disputes. It is the Supreme Court and High Court which look into the disputes. According to the Constitution of India, the Parliament consists of the Lok Sabha, the Rajya Sabha and the President of India.

147. Regarding the office of the Lok Sabha Speaker, consider the following statements:
  1. He/She holds the office during the pleasure of the President.
  2. He/She need not be a member of the House at the time of his/her election but has to become a member of the House within six months from the date of his/her election.
  3. If he/she intends to resign, the letter of his/her resignation has to be addressed to the Deputy Speaker.

Which of the statements given above is /are correct?

Explanation: B. If the Lok Sabha Speaker wants to resign, the letter of his / her resignation has to be addressed to the Deputy Speaker. The Speaker is elected by the Lok- Sabha from amongst its Members. Usually the Speaker remains in office during the life of the Lok-Sabha.

148. Which of the following can be said to be essentially the parts of 'Inclusive Governance'?
  1. Permitting the Non-Banking Financial Companies to do banking
  2. Establishing effective District Planning Committees in all the districts
  3. Increasing the government spending on public health
  4. Strengthening the Mid-day Meal Scheme

Select the correct answer using the codes given below :

Explanation: C. Inclusive governance Means that the benefit of governance Should reach the down trodden and to the last corner of the country.The Provisions given in option 2, 3 and 4 leads to the inclusive governance. Permitting the Non-Banking Financial companies to do banking is not directly linked to the inclusive governance.

149. In India, other than ensuring that public funds are used efficiently and for intended purpose, what is the importance of the office of the Comptroller and Auditor General (CAG)?
  1. CAG exercises exchequer control on behalf of the Parliament when the President of India declares national emergency/financial emergency.
  2. CAG reports on the execution of projects or programmes by the ministries are discussed by the Public Accounts Committee.
  3. Information from CAG reports can be used by investigating agencies to frame charges against those who have violated the law while managing public finances.
  4. While dealing with the audit and accounting of government companies, CAG has certain judicial powers for prosecuting those who violate the law.

Which of the statements given above is/are correct?

Explanation: C. Only 2nd and 3rd are correct statements.

150. The Prime Minister of India, at the time of his/her appointment

Explanation: A. Article 75(I) of the Indian Constitution provides that the Prime Minister shall be appointed by the President. The Constitution permits a person to be appointed PM without his\her being a member of either House of the Parliament at the time of appointment. However he/she has to become a Member of either house of parliament within Six Months from the date of his/her appointment as prime-minister.

151. A deadlock between the Lok Sabha and the Rajya Sabha calls for a joint sitting of the Parliament during the passage of
  1. Ordinary Legislation
  2. Money Bill
  3. Constitution Amendment Bill

Select the correct answer using the codes given below :

Explanation: A. Joint sitting is an extra-ordinary Machinery provided by the constitution to resolve a deadlock between the two houses over the passage of a bill. The provision of joint sitting is applicable to ordinary bills of financial bills only and not to Money bills or constitutional amendment bills.

152. With reference to consumers' rights/privileges under the provisions of law in India, which of the following statements is/are correct?
  1. Consumers are empowered to take samples for food testing.
  2. When a consumer files a complaint in any consumer forum, no fee is required to be paid.
  3. In case of death of a consumer, his/her legal heir can file a complaint in the consumer forum on his/her behalf.

Select the correct answer using the codes given below:

Explanation: C. Consumers are empowered to take samples for food testing. In case of death of a consumer his/her legal heir can file a complaint in the consumer forum on his /her behalf.

153. With reference to Indian History, the Members of the Constituent Assembly from the Provinces were

Explanation: C. The members of the constituent Assembly from the provinces were indirectly elected by the members of the provincial assemblies, who themselves were elected on a limited franchise.

154. According to the Constitution of India, which of the following are fundamental for the governance of the country?

Explanation: C. Directive Principles of State Policy are guidelines to the central and State government of India to be kept in mind while framing laws and policies. DPSPs aim to create social and economic conditions under which the citizens can lead a good life. They also aim to establish social and economic democracy through a welfare state. They act as a check on the government. It is a yardstick in the hands of the people to measure the performance of the government. It shall be the duty of the state to apply these principles in making laws.

155. Consider the following statements:
  1. An amendment to the Constitution of India can be initiated by an introduction of a bill in the Lok Sabha only.
  2. If such an amendment seeks to make changes in the federal character of the Constitution, the amendment also requires to be ratified by the legislature of all the States of India.

Which of the statements given above is/are correct?

Explanation: D. An amendment to the constitution of India can be initiated by an introduction of a bill in either house of the parliament. If such an amendment seeks to make changes in the federal character of the constitution, the amendment also requires to be ratified by the legislature of at least half of the states.

156. Which of the following bodies does not/do not find mention in the Constitution?
  1. National Development Council
  2. Planning Commission
  3. Zonal Councils

Select the correct answer using the codes given below.

Explanation: D. National Development council is not a constitutional body. It is an extra-constitutional body. Planning Commission is a non-constitutional and non-statutory body. It was created by the Govt. of India in 1950 by a resolution. Zonal councils were set up under the states Re-Organization Act, 1956 (So it is not a constitutional body) to faster inter-state cooperation and co-ordination among the states. Currently there are total five zonal councils viz Northern, Western, Eastern, Central and Southern.

157. Consider the following statements:
  1. National Development Council is an organ of the Planning Commission.
  2. The Economic and Social Planning is kept in the Concurrent List in the Constitution of India.
  3. The Constitution of India prescribes that Panchayats should be assigned the task of preparation of plans for economic development and social justice.

Which of the statements given above is/are correct?

Explanation: B. Directive Principles of State Policy lays down that the State shall take steps to organize village panchayats and endow them with such powers and authority as may be necessary to enable them to function as units of self government. Planning for economic development and social justice is one such power given to village panchayats. There are 52 items in the concurrent list. Economic and social planning is placed under entry no. 20. Article -40 of the constitution of India has the provision for organisation of vilage panchayats Directive Principles of state policy lays down that the state shall take steps to organise village panchayats and endow them with such powers and authority as may be necessary to enable them to function as units of self government. Accordingly village panchayats have been entrusted with the task of preparation of plans for economic development and social justice. The national development council includes members of the planning commission, but it is a separate body.

158. Economic Justice' as one of the objectives of the Indian Constitution has been provided in

Explanation: B. The Preamble to the Constitution of India in its introductory statement says- "Justice- Social, Economic and Political" and the Directive Principles of state policies aim to create social and economic condition under which the citizens can lead a good life. They also aim to establish social and economic democracy through a welfare state.

159. What will follow if a Money Bill is substantially amended by the Rajya Sabha?

Explanation: A. When a money bill returns to the Loksabha with amendments made by the Rajyasabha, it is open to Loksabha to accept or to reject any or all of the recommendations. When the Loksabha chooses to accept or decline the money bill with or without the recommendation, the money bill is deemed passed in both houses.

160. Which one of the following statements is correct?

Explanation: C. Correct option is C.

161. Consider the following statements : Attorney General of India can
  1. take part in the proceedings of the Lok Sabha
  2. be a member of a committee of the Lok Sabha
  3. speak in the Lok Sabha
  4. vote in the Lok Sabha

Which of the statements given above is/are correct?

Explanation: C. The Attorney General of India has a post parallel to any minister in Parliament. He can take part in the proceedings of either house. He can be a member of any committee of Parliament. He has the right to speak in the Parliament but he has no right to vote.

162. The Parliament can make any law for whole or any part of India for implementing international treaties

Explanation: D. Parliament has exclusive power to make law with respect to any of the matters enumerated with the Union List. According to entry no 14 in the Union List it reads- 'entering into treaties and agreements with foreign contries and implementing of treaties, agreement and convention with foreign countries'.

163. In the context of India, which of the following principles is/are implied institutionally in the parliamentary government?
  1. Members of the Cabinet are Members of the Parliament.
  2. Ministers hold the office till they enjoy confidence in the Parliament.
  3. Cabinet is headed by the Head of the State.

Select the correct answer using the codes given below.

Explanation: A. Minister/ministers can be removed by issuing no confidence motion in the parliament. All cabinet members are mandated by the constitution to be the member of either house of the parliament of India. The cabinet is headed by the prime minister and is advised by the cabinet secretary who also acts as the head of Indian Administrative service.

164. Consider the following statements:
  1. The Council of Ministers in the Centre shall be collectively responsible to the Parliament.
  2. The Union Ministers shall hold the office during the pleasure of the President of India.
  3. The Prime Minister shall communicate to the President about the proposals for legislation.

Which of the statements given above is/are correct?

Explanation: B. Article

165. Consider the following statements:
  1. The Chairman and the Deputy Chairman of the Rajya Sabha are not the members of that House.
  2. While the nominated members of the two Houses of the Parliament have no voting right in the presidential election, they have the right to vote in the election of the Vice President.

Which of the statements given above is/are correct?

Explanation: B. Correct option is B.

166. Under the Scheduled Tribes and Other Traditional Forest Dwellers (Recognition of Forest Rights) Act, 2006 who shall be the authority to initiate the process for determining the nature and extent of individual or community forest rights or both?

Explanation: D. Section C of the Forest Dwellers Act provides a transparent three step procedure for deciding on who gets rights. Firstly, the Gram Sabha makes a recommendation i.e, who has been cultivating land for how long, which minor forest produce is collected; etc. The Gram Sabha plays this role because it is a public body where all people participate and hence is fully democratic and transparent. The Gram Sabha's recommendation goes through two stages of screening committees- the Taluka and the District levels.

167. The Government enacted the Panchayat Extension to Scheduled Areas (PESA) Act in 1996. Which one of the following is not identified as its objective?

Explanation: D. D is correct option.

168. With reference to National Legal Services Authority, consider the following statements:
  1. Its objective is to provide free and competent legal services to the weaker sections of the society on the basis of equal opportunity.
  2. It issues guidelines for the State Legal Services Authorities to implement the legal programmes and schemes throughout the country.

Which of the statements given above is/are correct?

Explanation: C. The National Legal Services Authority (NALSA) has been constituted under the Legal Services Authorities Act, 1987 to provide free Legal Services to the weaker sections of the society and to organize Lok Adalats for amicable settlement of disputes. In every state, State Legal Services Authority has been constituted to give effect to the policies and directions of the NALSA and to give free legal services to the people and conduct Lok Adalats in the State.

169. Which one of the following Schedules of the Constitution of India contains provisions regarding anti-defection?

Explanation: D. The 52nd Amendment act of 1985, added 10th schedule to the Constitution. This is often referred to as anti-defection law.

170. In the Constitution of India, promotion of international peace and security is included in the

Explanation: B. Promotion of international peace and security is included in the Directive Principles of State Policy under Article 51 of constitution that mentions to promote international peace and security and maintain just and honourable relations between nations between nations; to foster respect for international law and treaty obligations, and to encourage settlements of international disputes by arbitration.

171. The power to increase the number of judges in the Supreme Court of India is vested in

Explanation: B. It is the Parliament which has the power to increase the number of judges in the Supreme Court of India. Parliament increased the number of judges from the original eight in 1950 to eleven in 1956, fourteen in 1960, eighteen in 1978, twenty-six in 1986 and thirty-one in 2008.

172. Consider the following statements regarding a No-Confidence Motion in India:
  1. There is no mention of a No-Confidence Motion in the Constitution of India.
  2. A Motion of No-Confidence can be introduced in the Lok Sabha only.

Which of the statements given above is/are correct?

Explanation: C. Article 75 says that the Council of Ministers shall be collectively responsible to Loksabha. In other words, Lok Sabha can remove them by passing a noconfidence motion. But the word "no confidence motion" itself is not mentioned in Constitution. It comes from Rule 198 of Lok Sabha Rules. Rajya Sabha cannot pass or remove council of ministers by passing no-confidence motion. No confidence motion can be introduced, only in Lok Sabha.

173. Which one of the following is the largest Committee of the Parliament?

Explanation: B. The largest Committee is the committee of Estimates, given its 30 members.

174. Which of the following are the discretionary powers given to the Governor of a State?
  1. Sending a report to the President of India for imposing the President's rule
  2. Appointing the Ministers
  3. Reserving certain bills passed by the State Legislature for consideration of the President of India
  4. Making the rules to conduct the business of the State Government

Select the correct answer using the code given below.

Explanation: B. The governor has Constitutional discretion in cases of Reservation of bill for consideration of the President and Recommendation of the imposition of President's rule. Therefore, statement 1 and 3 are definitely right. Moreover Governor only appoints those persons as ministers who are recommended by the Chief Minister. So Governor doesn

175. Which of the following is/are the function/functions of the Cabinet Secretariat?
  1. Preparation of agenda for Cabinet Meetings
  2. Secretarial assistance to Cabinet Committees
  3. Allocation of financial resources to the Ministries

Select the correct answer using the code given below.

Explanation: C. The functions of the Cabinet Secretariat includes preparation of agenda for Cabinet Meetings & Secretarial assistance to Cabinet Committees. However Allocation of financial resources to the Ministries as per the provisions in budget is the task of finance ministry.

176. Consider the following statements: A Constitutional Government is one which
  1. places effective restrictions on individual liberty in the interest of State Authority
  2. places effective restrictions on the Authority of the State in the interest of individual liberty

Which of the statements given above is/are correct?

Explanation: C. A constitutional Government needs to balance between individual liberty vis-a-vis State Authority.

177. The power of the Supreme Court of India to decide disputes between the Centre and the States falls under its

Explanation: C. The original jurisdiction of supreme court includes disputes between (i) The centre and one or more states; or (ii) The centre and any state or states on one side and one or more states on the other; or (iii) Between two or more states. In the above federal disputes, the Supreme Court has exclusive original jurisdiction. Meaning, no other count can decide such disputes.

178. The provisions in Fifth Schedule and Sixth Schedule in the Constitution of India are made in order to

Explanation: A. The Fifth Schedule of the Constitution of India deals with administration and control of scheduled areas and scheduled tribes in these areas. The Sixth Schedule to the Constitution of India contains provisions concerning the administration of tribal areas in the States of Assam, Meghalaya, Tripura and Mizoram.

179. Who/Which of the following is the custodian of the Constitution of India?

Explanation: D. The Supreme Court of India is the apex court in India. As stated by the Indian Constitution, the function of the Supreme Court of India is that of the custodian of the Constitution.

180. "To uphold and protect the Sovereignty, Unity and Integrity of India" is a provision made in the

Explanation: D. The idea behind incorporation of fundamental duties was to remind the citizens of the country that they have certain obligations towards the country and society. The fundamental duties were added to the constitution on the recommendations of the Swaran Singh Committee. There were ten fundamental duties at the time of incorporation but the eleventh was inserted by the 86th Amendment in 2002. To uphold and protect the sovereignty, unity and integrity of India is one of the fundamental duty mentioned in the constitution.

181. The ideal of Welfare State' in the Indian Constitution is enshrined in its

Explanation: B. Correct option is B.

182. Consider the following statements regarding the Directive Principles of State Policy:
  1. The Principles spell out the socio-economic democracy in the country.
  2. The provisions contained in these Principles are not enforceable by any court.

Which of the statements given above is / are correct?

Explanation: C. Correct option is C.

183. The Parliament of India acquires the power to legislate on any item in the State List in the national interest if a resolution to that effect is passed by the

Explanation: D. Correct option is D.

184. Consider the following statements:
  1. The Legislative Council of a State in India can be larger in size than half of the Legislative Assembly of that particular State
  2. The Governor of a State nominates the Chairman of Legislative Council of that particular State.

Which of the statements given above is/are correct?

Explanation: D. The maximum strength of the legislative council is fixed at one third of the total strength of the legislative assembly and the minimum strength is fixed at 40. The chairman of the legislative council is elected by the council itself from amongst its members.

185. With reference to the Union Government, consider the following statements:
  1. The Department of Revenue is responsible for the preparation of Union Budget that is presented to the Parliament.
  2. No amount can be withdrawn from the Consolidated Fund of India without the authorization from the Parliament of India.
  3. All the disbursements made from Public Account also need the authorization from the Parliament of India.

Which of the statements given above is / are correct?

Explanation: C. Correct option is C.

186. There is a Parliamentary System of Government in India because the

Explanation: D. The executive in a Parliamentary system is responsible to the legislature for all its actions. The ministers are answerable to the parliament and responsible to the Lok Sabha. The Council of Ministers remains in office as long as they enjoy the support and confidence of the Lok Sabha.

187. Consider the following statements:
  1. The Rajya Sabha has no power either to reject or to amend a Money Bill.
  2. The Rajya Sabha cannot vote on the Demands for Grants.
  3. The Rajya Sabha cannot discuss the Annual Financial Statement.

Which of the statements given above is / are correct?

Explanation: B. A Money Bill cannot be introduced in Rajya Sabha. Rajya Sabha has no power either to reject or amend a Money Bill. It can only make recommendations on the Money Bill. Rajya Sabha can discuss the budget but cannot vote on the demands for grants ( which is the exclusive privilage of the Lok- Sabha.

188. When a bill is referred to a joint sitting of both the Houses of the Parliament, it has to be passed by

Explanation: A. In India, if a bill has been rejected by any house of the parliament and if more than six months have elapsed, the President may summon a joint session for purpose of passing the bill. The bill is passed by a simple majority of a joint sitting. Joint sitting is an extra-ordinary machinery provided by the constitution to resolve a deadlock between the two houses over the passage of a bill. If the bill in dispute is passed by a majority of the total number of member both the houses present and voting in the joint sitting, the bill is deemed to have been passed by both the houses.

189. Consider the following statements
  1. The Executive Power of the Union of India is vested in the Prime Minister.
  2. The Prime Minister is the ex officio Chairman of the Civil Services Board.

Which of the statements given above is / are correct?

Explanation: D. The Executive powers of the Union of India is vested in the President. The Cabinet Secretary (and not the Prime Minister) is the ex-officio head of the Civil Services Board.

190. The fundamental object of Panchayati Raj system is to ensure which among the following?
  1. People's participation in development
  2. Political accountability
  3. Democratic decentralization
  4. Financial mobilization

Select the correct answer using the code given below.

Explanation: C. Financial mobilization is not the fundamental objective of PRls. Even though Panchayats have the powers to collect revenue.

191. The Parliament of India acquires the power to legislate on any item in the State List in the national interest if a resolution to that effect is passed by the

Explanation: D. If the Rajya Sabha declares that it is necessary in the national interest that Parliament should make laws on a matter in the State List, then the Parliament becomes competent to make laws on that matter. Such a resolution must be passed by the Rajya Sabha by a majority of not less than two-third of its members present and voting.

192. Consider the following statements:
  1. The minimum age prescribed for any person to be a member of Panchayat is 25 years.
  2. A Panchayat reconstituted after premature dissolution continues only for the remainder period.

Which of the statements given above is/are correct?

Explanation: B. The minimum age to contest Panchayat elections is 21 years.

193. With reference to the Gram Nyayalaya Act, which of the following statements is/are correct?
  1. As per the Act, Gram Nyayalayas can hear only civil cases and not criminal cases.
  2. The Act allows local social activists as mediators/ reconciliators.

Select the correct answer using the code given below.

Explanation: B. 2 only.

194. Which one of the following objectives is not embodied in the Preamble to the Constitution of India?

Explanation: B. Economic Liberty.

195. The mind of the makers of the Constitution of India is reflected in which of the following?

Explanation: A. Sir Alladi Krishnaswami Iyer, a member of the Constituent Assembly, had said 'The Preamble to our Constitution expresses what we had thought or dreamt so long."

196. Which of the following are envisaged by the Right against Exploitation in the Constitution of India?
  1. Prohibition of traffic in human beings and forced labour
  2. Abolition of untouchability
  3. Protection of the interests of minorities
  4. Prohibition of employment of children in factories and mines

Select the correct answer using the code given below:

Explanation: C. Correct option is C.

197. Which one of the following statements is correct?

Explanation: C. Rights are claims of the citizens against the State.

198. Which of the following statements is/are true of the Fundamental Duties of an Indian citizen?
  1. A legislative process has been provided to enforce these duties.
  2. They are correlative to legal duties.

Select the correct answer using the code given below:

Explanation: D. At present there are no laws for enforcing the the duty to develop scientific temper, humanism and the spirit of inquiry and reform or to enforce the fundamental duty to strive towards excellence. The list of fundamental duties does not cover other important duties like casting vote, paying taxes, family planning etc.

199. In the context of India, which one of the following is the correct relationship between Rights and Duties?

Explanation: A. Rights and duties are correlative and inseparable.

200. Which principle among the 'following was added to the Directive Principles of State Policy by the 42nd Amendment to the Constitution?

Explanation: B. The 42nd Amendment Act of 1976 added four new Directive Principles to the original list. They require the State: To secure opportunities for healthy development of children (Article 39); To promote equal justice and to provide free legal aid to the poor (Article 39 A); To take steps to secure the participation of workers in the management of industries (Article 43 A); To protect and improve the environment and to safeguard forests and wild life (Article 48 A).

201. Consider the following statements: With reference to the Constitution of India, the Directive Principles of State Policy constitute limitations upon
  1. legislative function.
  2. executive function.

Which of the above statements is/are correct?

Explanation: D. The phrase 'Directive Principles of State Policy' denotes the ideals that the State should keep in mind while formulating policies and enacting laws. These are the constitutional instructions or recommendations to the State in legislative, executive and administrative matters. Noted constitutional author Granville Austin considers DPSP as "positive obligations" of the state.

202. Democracy's superior virtue lies in the fact that it calls into activity

Explanation: A. The take-off point for a democracy is the idea of consent, i.e. the desire, approval and participation of people. It is the decision of people that creates a democratic government and decides about its functioning. So, since democracy requires voters' decision making- hence intelligence and character are called in.

203. One of the implications of equality in society is the absence of

Explanation: A. First step towards bringing about equality is of course ending the formal system of inequality and privileges. The caste system in India prevented people from the 'lower' castes from doing anything except manual labour. In many countries only people from some families could occupy high positions. Attainment of equality requires that all such restrictions or privileges should be brought to an end.

204. Which of the following are not necessarily the consequences of the proclamation of the President's rule in a State?
  1. Dissolution of the State Legislative Assembly
  2. Removal of the Council of Ministers in the State
  3. Dissolution of the local bodies

Select the correct answer using the code given below:

Explanation: B. when the President's Rule is imposed in a state, the President dismisses the state council of ministers headed by the chief minister. The state governor, on behalf of the President, carries on the state administration with the help of the chief secretary of the state or the advisors appointed by the President.

205. Which one of the following is not a feature of Indian federalism?

Explanation: D. Indian federation is not the result of an agreement among the states unlike the American federation.

206. Local self-government can be best explained as an exercise in

Explanation: B. Balwant Rai G Mehta Committee submitted its report in November 1957 and recommended the establishment of the scheme of 'democratic decentralisation', which ultimately came to be known as Panchayati Raj.

207. The main advantage of the parliamentary form of government is that

Explanation: C. the executive remains responsible to the legislature.

208. Out of the following statements, choose the one that brings out the principle underlying the Cabinet form of Government:

Explanation: B. mechanism for speeding up the activities of the government whose responsibilities are increasing day by day.

209. The Parliament of India exercises control over the functions of the Council of Ministers through.
  1. Adjournment motion
  2. Question hour
  3. Supplementary questions

Select the correct answer using the code given below:

Explanation: D. The Parliament exercises control over the ministers through various devices like question hour, discussions, adjournment motion, no confidence motion, etc. and Supplementary questions can be asked during the question hour.

210. With reference to the Parliament of India, consider the following statements:
  1. A private member's bill is a bill presented by a Member of Parliament who is not elected but only nominated by the President of India.
  2. Recently, a private member's bill has been passed in the Parliament of India for the first time in its history.

Which of the statements given above is/are correct?

Explanation: D. Private member's bill as a bill introduced by any member of the parliament who's not a minister. The Indian Express report in 2016 says only 14 private members bill have been passed since 1952.

211. For election to the Lok Sabha, a nomination paper can be filed by

Explanation: C. Correct option is C.

212. Consider the following statements:
  1. In the election for Lok Sabha or State Assembly, the winning candidate must get at least 50 percent of the votes polled, to be declared elected.
  2. According to the provisions laid down in the Constitution of India, in Lok Sabha, the Speaker's post goes to the majority party and the Deputy Speaker's to the Opposition.

Which of the statements given above is/are correct?

Explanation: D. Correct option is D.

213. Right to vote and to be elected in India is a

Explanation: C. Constitutional Right.

214. Consider the following statements:
  1. The Election Commission of India is a fivemember body.
  2. Union Ministry of Home Affairs decides the election schedule for the conduct of both general elections and by-elections.
  3. Election Commission resolves the disputes relating to splits/mergers of recognized political parties.

Which of the statements given above is/are correct?

Explanation: D. Election Commission has three election Commissioners. Election Commission decides the election schedule for the conduct of both general elections and bye-elections. It also decides the disputes relating to splits/mergers of recognized political parties.

215. In India, Judicial Review implies

Explanation: A. HC to examine the constitutionality of any law.

216. How is the National Green Tribunal (NGT) different from the Central Pollution Control Board
  1. The NGT has been established by an Act whereas the CPCB has been created by an executive order of the Government.
  2. The NGT provides environmental justice and helps reduce the burden of litigation in the higher courts whereas the CPCB promotes cleanliness of streams and wells, and aims to improve the quality of air in the country.

Which of the statement/s given above is/are correct:

Explanation: B. Central Pollution Control Board (CPCB), is statutory organization, was constituted in September, 1974 under the Water (Prevention and Control of Pollution) Act, 1974. Principal functions of the CPCB, as spelt out in the Water (Prevention and Control of Pollution) Act, 1974, and the Air (Prevention and Control of Pollution) Act, 1981, (i) to promote cleanliness of streams and wells in different areas. Environment: National Green Tribunal (NGT) was set up in 2010 under the NGT Act, 2010, for the purpose of effective and expeditious disposal of cases relating to environmental protection. The Tribunal shall not be bound by the procedure laid down under the Code of Civil Procedure, 1908, but shall be guided by principles of natural justice. The Tribunal is mandated to make an endeavor for disposal of applications or appeals finally within 6 months of filing.

217. With reference to the provisions made under the National Food Security Act, 2013 consider the following statements:
  1. The families coming under the category of 'below poverty line (BPL)' only are eligible to receive subsidised grains.
  2. The eldest woman in a household, of age 18 years or above, shall be the head of the household for the purpose of issuance of a ration card
  3. Pregnant women and lactating mothers are entitled to a take-home ration' of 1600 calories per day during pregnancy and for six months thereafter.

Which of the statements given above is/are correct?

Explanation: B. In 2013, NFSA was enacted to provide food security to 67% of the population in the form of highly subsidized foodgrains at Rs. 2 and 3 per Kg for wheat and rice respectively. The coverage under NFSA has been delinked from poverty estimates. Pregnant women and lactating mothers are entitled to a nutritious "take home ration" of 600 Calories and a maternity benefit of at least Rs 6,000 for six months.

218. Consider the following statements:
  1. As per the Right to Education (RTE) Act, to be eligible for appointment as a teacher in a State, a person would be required to possess the minimum qualification laid down by the concerned State Council of Teacher Education.
  2. As per the RTE Act, for teaching primary classes, a candidate is required to pass a Teacher Eligibility Test conducted in accordance with the National Council of Teacher Education guidelines.
  3. In India, more than 90% of teacher's education institutions are directly under the State Governments.

Which of the statements given above is/are correct?

Explanation: B. Under RTE Act section 23, National Council for Teacher Education (NCTE) decides the minimum qualification. After RTE it is mandatory that only those people may be appointed as teachers who are able to clear TET.

219. Consider the following statements
  1. The Food Safety and Standards Act, 2006 replaced the Prevention of Food Adulteration Act, 1954.
  2. The Food Safety and Standards Authority of India (FSSAI) is under the charge of Director General of Health Services in the Union Ministry of Health and Family Welfare.

Which of the statements given above is/are correct?

Explanation: A. After the formation of the FSS Act, 2006 a number of Acts and Orders that no longer served any purpose were repealed viz. The Edible Oils Packaging (Regulation) Order 1998, Fruit Products Order (FPO), 1955, Meat Food Products Order (MFPO), Prevention of Food Adulteration Act 1954 etc. While FSSAI acts under the Administrative control of Health ministry, FSSAI has an independent chairperson enjoying rank of Secretary to Government of India.

220. In the federation established by The Government on India Act of 1935. Residuary Power were given to the

Explanation: B. Residuary powers were in the hands of Governor General.

221. Consider the following statements :
  1. Aadhaar card can be used as a proof of citizenship or domicile.
  2. Once issued, Aadhaar number cannot be deactivated or omitted by the Issuing Authority.

Which of the statements given above is/are correct?

Explanation: D. Neither 1 nor 2.

222. Right to Privacy is protected as an intrinsic part of Right to Life and Personal Liberty. Which of the following in the Constitution of India correctly and appropriately imply the above statement?

Explanation: C. The Supreme Court ruled that "the right to privacy is protected as an intrinsic part of the right to life and personal liberty under Article 21 and as a part of the freedoms guaranteed by Part III of the Constitution". Article 14- Gives the Right to Equality. 42nd Constitutional Amendment Act 1976, is known as mini constitution. Article 17- Related to the Abolition of Untouchability. It is part of Right to Equality. Part IV- Directive Principles of State Policy, does not have any mention about the Privacy. Article 24- Prohibition of employment of children in factories, et(c) 44th Constitution Amendment- 44th amendment of the Constitution was enacted by the Janata Government mainly to nullify some of the amendments made by the 42nd Amendment Act, 1976.

223. With reference to the election of the President of India, consider the following statements:
  1. The value of the vote of each MLA varies from State to State.
  2. The value of the vote of MPs of the Lok Sabha is more than the value of the vote of MPs of the Rajya Sabha

Which of the statements given above is/are Correct?

Explanation: C. Value of 1 MLA's vote is based on total population of state to be divided by the total MLAs. Hence it ought to vary from state to state. While the value of an MLA's vote depends on the population of the state he or she belongs to, the value of an MP's vote remains the same at 708.

224. Consider the following statements:
  1. No criminal proceedings shall be instituted against the Governor of a State any court during his term of office.
  2. The emoluments and allowances of the Governor of a State shall not be diminished during his term of office.

Which of the statements given above is/are correct?

Explanation: C. Governor enjoys personal immunity from legal liability for his official acts. During his term of office, he is immune from any criminal proceedings, even in respect of his personal acts. He cannot be arrested or imprisoned. His emoluments and allowances cannot be diminished during his term of office. Thus both statements are right.

225. With reference to the Parliament of India, which of the following Parliamentary Committees scrutinizes and reports to the House whether the powers to make regulations, rules, sub-rules, by-laws, etc conferred by the Constitution or delegated by the Parliament are being properly exercised by the Executive within the scope of such delegation?

Explanation: B. Committee on Subordinate legislation examines and reports to the House whether the powers to make regulations, rules, sub-rules and bye-laws delegated by the Parliament or conferred by the Constitution to the Executive are being properly exercised by it. In both the Houses, the committee consists of 15 members. It was constituted in 1953.

226. Consider the following statements:
  1. The Speaker of the Legislative Assembly shall vacate his/her office if he/she ceases to be a member of the Assembly.
  2. Whenever the Legislative Assembly is dissolved, the Speaker shall vacate his/her immediately.

Which of the statements given above is/are correct?

Explanation: A. Correct option is A.

227. If the President of India exercises his power as provided under Article 356 of the Constitution in respect of a particular State, then

Explanation: B. The President's Rule can be proclaimed under Article 3. He can declare that the powers of the state legislature are to be exercised by the Parliament.

228. Consider the following statements :
  1. The Parliament of India can place a particular law in the Ninth Schedule of the Constitution of India
  2. The validity of a law placed in the Ninth Schedule cannot be examined by any court and no judgement can be made on it.

Which of the statements given above is/are correct ?

Explanation: A. the 1st Amendment to the constitution (1951) had inserted new Articles 31A and 31B and the Ninth Schedule, thus securing the constitutional validity of zamindari abolition laws by, among other things, specifying that they could not be challenged on the grounds that they violated the Fundamental Rights. Subsequent governments have added other progressive laws in the same, to give them immunity from litigation. Government added the acts in the 9th schedule, 'via parliament'. In I.R. Coelho case (2007), the Supreme Court ruled that there could not be any blanket immunity from judicial review of laws included in the Ninth Schedule.

229. Consider the following statements:
  1. In the first Lok Sabha, the single largest party in the opposition was the Swatantra Party.
  2. In the Lok Sabha, a "Leader of the Opposition" was recognized for the first time in 1969.
  3. In the Lok Sabha, if a party does not have a minimum of 75 members, its leader cannot be recognized as the Leader of the Opposition.

Which of the statements given above is/are correct?

Explanation: B. The results of the first general election did not surprise anyone

230. Which one of the following reflects the nicest, appropriate relationship between law and liberty?

Explanation: B. Correct option is B.

231. Which of the following are regarded as the main features of the "Rule of Law"?
  1. Limitation of powers
  2. Equality before law
  3. People's responsibility to the Government
  4. Liberty and civil rights

Select the correct answer using the code given below:

Explanation: C. The idea of rule of law implies that all individuals - rich and poor, men or women, forward or backward castes - are subjected to the same law. The principal role of the judiciary is to protect rule of law and ensure supremacy of law. It safeguards rights of the individual

232. Consider the following statements:
  1. The 44 Amendment to the Constitution of India introduced an Article placing the election of the Prime Minister beyond judicial review.
  2. The Supreme Court of India struck down the 99th Amendment to the Constitution of India as being violative of the independence of judiciary.

Which of the statements given above is/are correct?

Explanation: B. In 1975, Parliament passed the Thirty-ninth amendment to the Constitution which removed the authority of the Supreme Court to adjudicate petitions regarding elections of the President, Vice President, Prime Minister and Speaker of the Lok Sabha. Instead, a body constituted by Parliament would be vested with the power to resolve such election disputes.

233. Consider the following statements:
  1. The motion to impeach a Judge of the Supreme Court of India cannot be rejected by the Speaker of the Lok Sabha as per the Judges (Inquiry) Act 1968.
  2. The Constitution of India defines and gives details or what constitutes
  3. The details of the process of impeachment of the Judges of the Supreme Court of India are given in 4 the Judges (Inquiry) Act, 1968.
  4. If the motion for the impeachment of a Judge is taken up for voting, the law requires the motion to be backed by each House of the Parliament and supported by the majority of total membership of that House and by not less than two-thirds of total members of that House present and voting.

Which of the statements given above is/are correct?

Explanation: C. 3 and 4 only.

234. The Ninth Schedule was introduced in the Constitution of India during the prime ministership of

Explanation: A. Jawaharlal Nehru

235. Consider the following statements:
  1. The Parliament (Prevention of Disqualification) Act, 1959 exempts several posts from disqualification on the grounds of 'Office of Profit'.
  2. The above-mentioned Act was amended five times.
  3. The term 'Office of Profit' is well-defined in the Constitution of India.

Which of the statements given above is/are correct?

Explanation: A. 1 and 2 only.

236. Under which Schedule of the Constitution of India can the transfer of tribal land to private parties for mining be declared null and void?

Explanation: B. Fifth Schedule.

237. With reference to the Constitution of India, prohibitions or limitations or provisions contained in ordinary laws cannot act as prohibitions or limitations on the constitutional powers under Article 142. It could mean which one of the following?

Explanation: B. The Supreme Court of India is not constrained in the exercise of its powers by the laws made by Parliament.

238. With reference to the Legislative Assembly of a State in India, consider the following statements:
  1. The Governor makes a customary address to Members of the House at the commencement of the first session of the year.
  2. When a State Legislature does not have a rule on a particular matter, it follows the Lok Sabha rule on that matter.

Which of the statements given above is/are correct?

Explanation: A. Article 176(1) of the Constitution of India enjoins that the Governor shall Address both the Houses assembled together at the commencement of the first Session after each general election to the Assembly and at the commencement of the first session of each year and inform the Legislature of the causes of its Summons. In case of absence of rules in particular matter, the speaker of state legislative assembly/council decides rules.

239. Which Article of the Constitution of India safeguards one's right to marry the person of one's choice?

Explanation: B. Article 21.

240. Which one of the following suggested that the Governor should be an eminent person from outside the State and should be a detached figure without intense political links or should not have taken part in politics in the recent past?

Explanation: C. Sarkaria commission gave its recommendations for the selection of Governors.

241. Consider the following statements:
  1. As per law, the Compensatory Afforestation Fund Management and Planning Authority exists at both National and State levels.
  2. People's participation is mandatory in the compensatory afforestation programmes carried out under the Compensatory Afforestation Fund Act, 2016.

Which of the statements given above is/are correct?

Explanation: A. The Compensatory Afforestation Fund Act 2016, establishes the National Compensatory Afforestation Fund under the Public Account of India, and a State Compensatory Afforestation Fund under the Public Account of each state.these funds are to be managed by the National and State Compensatory Afforestation Fund Management and Planning Authorities, also set up under the Act. However, there is no mention in the Act of mandatory people's participation in carrying out programmes under the Act.

242. In India, which of the following review the independent regulators in sectors like telecommunications, insurance, electricity, etc.?
  1. Ad Hoc Committees set up by the Parliament
  2. Parliamentary Department Related Standing Committees
  3. Finance Commission
  4. Financial Sector Legislative Reforms Commission

Select the correct answer using the code given below:

Explanation: A. 1 and 2.

243. Consider the following statements:
  1. Petroleum and Natural Gas Regulatory Board (PNGRB) is the first regulatory body set up by the Government of India.
  2. One of the tasks of PNGRB is to ensure competitive markets for gas.
  3. Appeals against the decisions of PNGRB go before the Appellate Tribunals for Electricity.

Which of the statements given above are correct?

Explanation: B. The Petroleum and Natural Gas Regulatory Board (PNGRB) was constituted under The Petroleum and Natural Gas Regulatory Board Act, 2006. The independent regulator TRAI is the first independent regulator in India.

244. With reference to the Constitution of India, consider the following statements:
  1. No High Court shall have the jurisdiction to declare any central law to be constitutionally invalid.
  2. An amendment to the Constitution of India cannot be called into question by the Supreme Court of India.

Which of the statements given above is/are correct?

Explanation: D. Neither 1 nor 2.

245. In the context of polity, which one of the following would you accept as the most appropriate definition of liberty?

Explanation: B. Absence of restraint.

Question No.
Toppers

Top Popular Courses


Newsletter Subscription
SMS Alerts

Important Links

UPSC GS Mains Crash Course - RAW Prelims Answer Key 2024